0% found this document useful (0 votes)
433 views11 pages

Limsup Questions 1

The document provides solutions to 6 questions: 1) Proves an inequality relating lim sup of sums and individual lim sups. Provides an example of strict inequality. 2) Proves the lim sup of a sequence is equal to the supremum of a related set S. Also proves the analogous result for lim inf. 3) Finds the lim sup and lim inf of a recursively defined sequence by deriving a formula for the terms and analyzing the behavior. 4) Proves if every subsequence converges to a, then the full sequence converges to a. Also proves an analogous result replacing "subsequence" with a stronger condition. 5) Proves a sequence is Cauchy (

Uploaded by

Rishabh Sarma
Copyright
© © All Rights Reserved
We take content rights seriously. If you suspect this is your content, claim it here.
Available Formats
Download as PDF, TXT or read online on Scribd
0% found this document useful (0 votes)
433 views11 pages

Limsup Questions 1

The document provides solutions to 6 questions: 1) Proves an inequality relating lim sup of sums and individual lim sups. Provides an example of strict inequality. 2) Proves the lim sup of a sequence is equal to the supremum of a related set S. Also proves the analogous result for lim inf. 3) Finds the lim sup and lim inf of a recursively defined sequence by deriving a formula for the terms and analyzing the behavior. 4) Proves if every subsequence converges to a, then the full sequence converges to a. Also proves an analogous result replacing "subsequence" with a stronger condition. 5) Proves a sequence is Cauchy (

Uploaded by

Rishabh Sarma
Copyright
© © All Rights Reserved
We take content rights seriously. If you suspect this is your content, claim it here.
Available Formats
Download as PDF, TXT or read online on Scribd
You are on page 1/ 11

Solutions to Assignment-2

Only submit the questions in red.

1. (a) For any two sequences {an } and {bn } show that

lim sup(an + bn ) ≤ lim sup an + lim sup bn ,


n→∞ n→∞ n→∞

unless the right hand side is of the form ∞ − ∞.

Solution: Assume both the limsups are finite (the other cases are also similar). Let A =
lim supn→∞ an , B = lim supn→∞ bn and L = lim supn→∞ (an + bn ). Suppose L > A + B.
Choose an ε > 0 such that L − ε > A + B + ε. For any N > 0 there exists an n > N such
that
an + bn > L − ε. (0.1)
On the other hand, there exists N1 such that for all n > N1 ,
ε
an < A + ,
2
and there exists N2 such that for all n > N2 ,
ε
bn < B + .
2
But then if N = max(N1 , N2 ), then for any n > N we have

an + bn < A + B + ε < L − ε,

contradicting (0.1).

(b) Find sequences {an } and {bn } with strict inequality above.

Solution: Let an = (−1)n and bn = (−1)n−1 . Then an +bn = 0 for all n, and so lim supn→∞ (an +
bn ) = 0 while, lim supn→∞ an + lim supn→∞ bn = 1 + 1 = 2.

2. Let {an } be a sequence of real numbers, and let


k→∞
S = {x ∈ R | ∃ a sub-sequence ank such that ank −−−−→ x}.

(a) Show that L = lim sup an if and only if L = sup S.

Solution: Suppose L = lim sup an . First, we claim that L ∈ S. To see this, note that by the
equivalent characterization of limsup, there exists n1 such that

an1 > L − 1.

1
Given n1 , there exists n2 > n1 such that
1
an2 > L − .
2
Having chosen n1 < n2 < · · · < nk−1 , let nK > nk−1 such that
1
ank > L − .
k
k→∞
Claim. ank −−−−→ L.
Proof. Let ε > 0. Then there exists N such that for all n > N ,

an < L + ε.
k→∞
Since nk −−−−→, there exists a K1 such that for all k > K1 , nk > N . In particular, for all
k > K1 ,
ank < L + ε.
Let K2 such that 1/K2 < ε. Then by our choice of the subsequence ank , for all k > K2 ,

1 1
ank > L − >L− > L − ε.
k K2
In particular, if K = max(K1 , K2 ), and k > K then

|ank − L| < ε,
k→∞
and hence ank −−−−→ L.
This shows that L ∈ S. In particular, L ≤ sup S. Suppose L < sup S. Let ε > 0 such that
L + ε < sup S. There exists an N such that for all n > N ,

an < L + ε,

and so for any x ∈ S, x < L + ε. Taking sup,

sup S ≤ L + ε,

a contradiction. Hence L = sup S.

(b) Formulate and prove the analogous statement for lim inf.

Solution: The corresponding statement would be

lim inf an = inf S.


n→∞

One can argue as above, or alternately, use the standard trick that if bn = −an , then

lim inf an = − lim sup bn .


n→∞ n→∞

Note. From now on, you can use the conclusions of this exercise as a theorem. So now, you have a
definition of lim sup and two other equivalent characterizations.

2
3. Find the lim sup and lim inf of the sequence {an } defined recursively by
a2m−1 1
a1 = 0, a2m = , a2m+1 = + a2m .
2 2
Justify your answers with complete proofs.

Solution: To get some intuition, we compute the first few terms of the sequence,
1 1 3 3 7 7 15
a2 = 0, a3 = , a4 = , a5 = , a6 = , a7 = , a8 = , a9 = .
2 4 4 8 8 16 16
Seeing a pattern, we make the claim -
1
Claim. lim inf n→∞ an = 2 and lim supn→∞ an = 1.
Proof. The easiest proof is to simply find a formula for the nth term. We claim that
( m
2 −1
an = 2m−12m , n = 2m + 1
−1
2m , n = 2m.

We prove this by induction. The base cases n = 1 are seen to be true. Suppose the formula is
correct for some n = 2m − 1 = 2(m − 1) + 1. We then prove the formula for 2m and 2m + 1.

a2m−1 2m−1 − 1
a2m = = .
2 2m
But then again by the recursion formula,

1 1 2m−1 − 1 2m − 1
a2m+1 = + a2m = + m
= .
2 2 2 2m
Once we have the formula, note that{a2m+1 } is a increasing to 1 and {a2m } is a sequence increasing
N −1
to 1/2. Then clearly, uN = sup{ak | k > N } = 1, and lN = inf{ak | k > N } > 2 2N −1 . Letting
N → ∞, we complete the proof of the claim.

4. (a) Let {an } be a bounded sequence with the property that every convergent subsequence converges to
the same limit a. Show that the entire sequence {an } converges and limn→∞ an = a.

Solution: If not, then there exists an ε > 0 and a subsequence bk = ank such that

|bk − a| > ε

for all k. By Bozlano-Weierstrass, since {bk } is bounded, there exists a further sub-sequence
bkj which converges. But bkj = ankj is also a sub-sequence of an and since it converges, by
the hypothesis, it must converge to a. But by our choice of {bk }, |bkj − a| > ε for all j, a
contradiction.

(b) Now assume that {an } is a sequence with the property that every subsequence has a further sub-
sequence that converges to the same limit a. Show that the entire sequence {an } converges and
limn→∞ an = a.

Solution: If not, then there is an ε > 0 and a sub-sequence bk = ank such that |bk − a| > ε.
By hypothesis, bk has a subsequence, say {bkj }, that converges to a. But then
lim |bkj − a| = 0,
j→∞

3
which contradicts the fact that |bkj − a| > ε.

5. Let {an }∞
n=0 be a sequence of real numbers satisfying

1
|an+1 − an | ≤ |an − an−1 |.
2
Show that the sequence converges. Hint. Show that the sequence is Cauchy.

Solution: Inductively, we see that for any natural number k,


1
|ak+1 − ak | ≤ |a1 − a0 |.
2k
Now if m > n then by triangle inequality

|am − an | = |am − am−1 + am−1 − am−2 + am−2 − · · · − an |


≤ |am − am−1 | + |am−1 − am−2 | + · · · + |an+1 − an |
m−1
X 1
≤ |a1 − a0 |
2k
k=n

|a1 − a0 | X 1

2n 2k
k=0
≤ 2−n |a1 − a0 |.

Given ε > 0, let N such that 2−N |a1 − a0 | < ε. Then for any m > n > N , |am − an | < ε, and the
sequence is Cauchy.

6. Let S = {n1 , n2 , · · · } denote the collection of those positive integers that do


P∞ not have the digit 0 in their
decimal representation. (For example 7 ∈ S but 101 ∈ / S). Show that k=1 1/nk converges. Note.
This should be a surprising result in that leaving out only a few (but of course still infinite) terms out
of the harmonic series, we end up with a series that suddenly converges.

Solution: Consider the one-digit numbers in S, namely {1, 2, · · · , 9}. Since each is bigger than one,
the sum of reciprocals is
1 1
1 + · · · + < 9.
2 9
Next, consider the two-digit numbers in S. There are 81 of them, and each is bigger than 10, and
so the sum of reciprocals satisfies the estimate,
1 1 1 1 1 1 81
+ + ··· + + + ··· + + < .
11 12 19 21 98 99 10
In general, consider the subset Sk of numbers in S with k digits, that is numbers between 10k and
10k+1 . The number of such numbers is 9k . That is because there are k digits, and each digit has 9
options. Moreover, all these numbers are bigger than 10k , and so
X 1 9k+1
< .
n 10k
n∈Sk

4
Summing over the reciprocals of numbers with at-most m-digits,
m X m ∞
X 1 X 9k 9 X 9k 81 1
<9 k
< k
< 9 = 81.
n 10 10 10 10 1 − 10
k=1 n∈Sk k=1 k=0
P∞
In particular the partial sums of k=1 1/nk are bounded by 81 and since the terms in the series are
positive by the monotone convergence theorem, the series converges.

7. The Fibonacci numbers {fn } are defined by

f0 = f1 = 1, and fn+1 = fn + fn−1 for n = 1, 2, · · · .

For n = 1, 2, · · · , we also define rn = fn+1 /fn .


(a) Find a formula for rn+1 in terms of rn . Dividing the above recurrence by fn , we obtain
1
rn = 1 + ,
rn−1
or
1
rn+1 = 1 + .
rn
(b) Show that fn ≥ n for all n ≥ 2.

Solution: Easy proof by induction.

(c) Show that fn+1 fn−1 − fn2 = (−1)n+1 .

Solution: We proceed by induction. For n = 1, fn+1 fn−1 − fn2 = f2 f0 − f12 = 2 · 1 − 11 = 1 =


(−1)1+1 , and so the identity is verified. Suppose the identity is verified for n − 1, that is we
2
have fn fn−2 − fn−1 = (−1)n . Then

fn+1 fn−1 − fn2 = (fn + fn−1 )fn−1 − fn2


2
= fn−1 + fn (fn−1 − fn )
2
= fn−1 − fn fn−2 = −(−1)n = (−1)n+1 .

(d) Hence show that if n ≥ 2, then


1
|rn+1 − rn | ≤ .
(n − 1)2

Solution: Note that


f fn 1
n+1
|rn − rn−1 | = − =
fn fn−1 fn fn−1

by the identity. By part (b), fn > n for all n ≥ 2 and so the proof is completed by the
elementary observation that (n − 1)2 < n(n − 1).

(e) Hence show that the sequence of ratios {rn } converge, and compute it’s limit. Note. This limit is
the so-called golden ratio.

5
Solution: For any n < m, by the triangle inequality and part(d),
1 1 1
|rm − rn | ≤ + + ··· .
n2 (n + 1)2 (m − 1)2

Since the right hand side is the tail end of a converging series, by the Cauchy criteria, for any
ε > 0, there exists an N such that for any n, m > N , the right hand side can be made smaller
than ε. This shows that {rn } is Cauchy, and hence converges. To find the actual limit, first note
that if L = limn→∞ rn , then L 6= 0. letting n → ∞ on both sides of the recurrence obtained in
part(a) we obtain
1
L=1+ .
L

Solving the quadratic L2 − L − 1 = 0, we see that the roots are (1 ± 5)/2, of which the only
positive root has to be L.

8. Investigate the behavior of each series (convergence, divergence, conditional convergence, absolute con-
vergence). In cases that there is a parameter (p, q or r) find the range of values where the series exhibits
the above behavior.
P∞ n p P∞ n!
1. n=1 p n (p > 0) 4. n=1 nn
P∞ √
n+1−

n
P∞ √ n
n=1 ( n − 1)
n 5. n
2. n=1 (−1) np
P∞ 1
P∞ 1
3. n=1 pn −q n , (0 < q < p) 6. n=1 1+r n .

Solution:
√ n→∞
1. Let an = pn np . Then n an = pnp/n −−−−→P p. So by the root test, the series converges if p < 1
and diverges if p > 1. If p = 1, the series n clearly diverges.
√ √
n+1− n
2. Let an = (−1)n np . Then

1
|an | = √ √ .
np ( n + 1 + n)
1
Comparing this with n−(p+ 2 ) , we see that
P
an converges absolutely for p > 1/2. On the
other Phand when
√ p < 0,
√ clearly
√ the series diverges. At p = 0, the partial sums of the series are
n
sn = k=1 k + 1 − k = n + 1 − 1 which clearly diverge. For p ∈ (0, 1/2] the series does
not converge

absolutely.

To check for conditional convergence we apply alternating series test.
Let bn = n+1− np
n
= √ 1 √ , and hence decreases to 0 if p > 0. So by the alternating
np ( n+1+ n)
series test the series converges conditionally in the range p ∈ (0, 1/2].
3. We can write
1 1
an = = n .
pn − q n p (1 − (q/p)n )
Since q < p, limn→∞ (q/p)n = 0, and so there exists an N such that for all n > N , (q/p)n < 1/2
or (1 − (q/p)n )−1 < 2. On the other hand, for any n, (1 − (q/p)n )−1 > 1, and so for n > N ,
1 2
< an < n .
pn p
By comparison test the series converges if p > 1 and diverges if 0 < p ≤ 1.

6
4. We use the ratio test. If aN = nn /n!, then

an+1 (n + 1)n+1 (n + 1)n  1 n n→∞


= n = = 1 + −−−−→ e > 1,
an n (n + 1) nn n

and so the series diverges.



5. We use root test. Let an = ( n n − 1)n . Then
√ √ n→∞
n
an = n
n − 1 −−−−→ 0 < 1,

and so the series converges.


6. Let an = (1 + rn )−1 . If |r| ≤ 1, then {an } does not converge to zero, and so the series diverges.
If |r| > 1, limn→∞ r−n = 0, and so there exists an N ∈ N such that 1 + r−n > 1/2 for all
n > N (note that r could be negative, or else 1 + r−n is of course bigger than 1). Then for
n > N,
|r|−n 2
|an | = −n
< n,
1+r |r|
and so by limit comparison test, the series is absolutely convergent for |r| > 1.

9. (a) Let {an } be a sequence of of positive real numbers. Show that


an+1 √ √ an+1
lim inf ≤ lim inf n an ≤ lim sup n an ≤ lim sup .
n→∞ an n→∞ n→∞ n→∞ an

You may assume that each of the quantities is finite, even though the result holds true for ex-
tended reals. Hint. Proceed by contradiction. √ For instance, for the rightmost inequality, let
U = lim supn→∞ aan+1 n
and L = lim supn→∞
n
a n and suppose L > U . Then use the equivalent
characterizations of lim sup to draw a contradiction.

Solution: We will show that


√ an+1
lim sup n
an ≤ lim sup .
n→∞ n→∞ an

The other inequalities also follow in a similar fashion. Denote L = lim supn→∞ n an and
an+1
U = lim supn→∞ an . We proceed by contradiction, so suppose L > U . Let β ∈ (U, L). Then
there exists an N such that for all n > N ,
a
n+1
< β,
an

or equivalently, |an+1 | < β|aN |. Inductively, one can conclude that |an | < β n−N |aN |. That is,
for any n > N ,
|an |1/n < β 1−N/n |aN |1/n .
Now N is fixed, so taking limsup on both sides, since limn→∞ β 1−N/n |aN |1/n = β, we see that

lim sup |an |1/n ≤ β,


n→∞

which is a contradiction.
P P
(b) Show that if an converges by the ratio test, then an also converges by the root test.

7
P
Solution: If an converges by the ratio test, then lim supn→∞ |apn+1 /an | < 1. But then by
the above set of inequalities, we necessarily have that lim supn→∞ n |an | < 1, and so the series
also converges by the root test.

(c) Consider the sequence {an }∞


n=0 ,
(
1
an =
1
= 2n−1 , n is odd
n+(−1)n
2 1
2n+1 , n is even.
p
Compute (with proper justifications) lim sup n |an | and lim sup |an+1 /an |. Show that the series
converges by the root test. Does the ration test work?

Solution: Note that


p
n 1 n→∞ 1
|an | = −−−−→ ,
21+(−1)n /n 2
p
and so lim sup n
|an | = 1/2. On the other hand,
(
1
an+1 , n is odd
= 8
an 2, n is even,
p
and so lim sup |an+1 /an | = 2 P
and lim inf |an+1 /an | = 1/8. Since lim sup n
|an | = 1/2 < 1, the
root test says that the series an converges. On the other hand since
a a
n+1 n+1
lim inf < 1 < lim sup
an an
,

the ratio test is inconclusive.

(d) Let bn = nn /n!. Show that p


n
lim bn = e.
n→∞
Hint. It is easier to compute the limiting ratios.

Solution: Note that


an+1 (n + 1)n+1 n! (n + 1)n+1 (n + 1)n  1 n n→∞
= n
= n
= n
= 1+ −−−−→ e.
an (n + 1)!n (n + 1)n n n

So lim inf n→∞ aan+1 = lim supn→∞ aan+1 = e. But then by the chain of inequalities in the first
n n √ √
part the middle two terms are also equal, that is, lim inf n→∞ n an = lim supn→∞ n an = e,
and so

lim n an = e.
n→∞

P
10. (a) Show that if an > 0, and limn→∞ nan = l 6= 0, then an diverges.

Solution: Since nan → l 6= 0, applying the definition of convergence with ε = |l|/2 > 0, there
exists N ∈ N such that
|l|
n > N =⇒ |nan − l| < .
2
P
In particular,
P for n > N , an > |l|/2n. By the comparison test, since 1/n diverges, it follows
that an also diverges.

8
P P p
(b) Given that an converges
P absolutely, show that an also converges whenever p > 1. Give a
counterexample, if an only converges conditionally.
P
Solution: Since an converges absolutely, by the divergence test, |an | → 0. In particular,
Pn > pN , |an | < 1. But then P
there exists N such that for all for any p > 1, |an |p < |an | when
n > NP . By comparison test, |an | converges, and hence apn also converges.
√ This is not
true if an only converges conditionally. For instance, consider an = (−1)n / n and p = 2.

11. Consider each of the following propositions. Provide short proofs for those that are true and counterex-
amples for any that are not.
P P
(a) If an converges and the sequence {bn } also converges, then an bn converges.
P P 2
(b) If an converges conditionally, then n an diverges.
P 2
Solution: The proposition is true. If not, then n an converges, and soPlimn→∞ n2 an = 0.
In particular, there exists N such that for all n > N , |an | < 1/n2 , and so an must converge
absolutely by the comparison test. A contradiction!
P
(c) If {an } is a decreasing sequence, and an converges, then limn→∞ nan = 0.
P
Solution: The proposition is true. Since an is convergent, limn→∞ an = 0. But then since
an is also decreasing, it follows that an ≥ 0. By the Cauchy criteria, given any ε > 0, there
exists an N such that for all n > m > N ,
n
X ε
ak < .
2
k=m

Applying this to m = bn/2c with n > 2N , and using the fact that an decreases
n
ε X nan
> ak > .
2 2
k=bn/2c

So given ε > 0, if n > 2N , then |nan | < ε, and hence limn→∞ nan = 0.

12. (a) For any n ∈ N, show that the function pn (x) = xn is continuous on all of R. Show the explicit
dependence of δ on ε and the point that you are looking at.

Solution: We prove continuity at x = a. Let ε > 0 be given. We need to estimate

|pn (x) − pn (a)| = |xn − an |


= |x − a||xn−1 + xn−2 a + · · · + an−1 |.

Now, |x − a| < δ, where δ > 0 is to be chosen. Suppose, we choose δ < 1, then clearly
|x| < |a| + 1. A general term on the right is of the form xj an−1−j for j = 0, · · · , n − 1. So if
δ < 1, we have
|xj an−1−j | < (|a| + 1)j |a|n−1−j < (|a| + 1)n−1 .
Then by triangle inequality,

|xn−1 + xn−2 a + · · · + an−1 | < n(1 + |a|)n−1 .

9
So if δ < 1 and |x − a| < δ, then

|pn (x) − pn (a)| < nδ(1 + |a|)n−1 .

Our aim is to make this smaller than ε, and so simply choose


ε
δ< .
n(1 + |a|)n−1

Together with δ < 1, we see that if


 ε 
δ = min 1, ,
n(1 + |a|)n−1

then
|x − a| < δ =⇒ |pn (x) − pn (a)| < ε.


(b) Show that f (x) = x is continuous on (0, ∞).

Solution: Let ε > 0, and |x − a| < δ for some δ > 0 to be chosen later. Clearly

√ √ |x − a| δ
| x − a| = √ √ < √ √ .
| x + a| | x + a|

Now, a > 0, by choosing δ < a/2,

|x − a| < δ =⇒ x > a/2.

So √ √ √
x+ a > 3 a/2,
and
√ √ 2δ
| x − a| < √ .
3 a
So simply pick √
3 a 
a
δ = min , ε .
2 2

(c) Show that fn (x) = x1/n is continuous on (0, ∞).

Solution: Again, we prove continuity at x = a. Let ε > 0. From the identity, we see that

x − a = (x1/n − a1/n )(x1−1/n + x1−2/n a1/n · · · + a1−1/n )

Then
|x − a|
|fn (x) − fn (a)| = .
|x1−1/n + x1−2/n a1/n · · · + a1−1/n |
Again as before, since a > 0, if δ < a/2, then x > a/2 and so

x1−1/n + x1−2/n a1/n · · · + a1−1/n > na1−1/n cn ,

where cn is the constant (independent of a)

cn = 21−1/n + 21−2/n + · · · + 1.

10
And so if |x − a| < δ and δ < a/2 we have

δ
|fn (x) − fn (a)| < .
ncn a1−1/n
So simply pick a 
δ = min , ncn a1−1/n ε .
2

Hint. For all parts the following identity might be useful.

an − bn = (a − b)(an−1 + an−2 b + · · · + abn−2 + bn−1 ).

11

You might also like